Difference between revisions of "2025 SSMO Speed Round Problems/Problem 2"
(Created page with "==Problem== Let <math>A</math> and <math>B</math> be points such that <math>AB = 50</math>. Points <math>M</math> and <math>N</math> lie on <math>\overline{AB}</math> such th...") |
(→Solution) |
||
Line 4: | Line 4: | ||
==Solution== | ==Solution== | ||
+ | |||
+ | <asy> | ||
+ | pair A,B,M,N; | ||
+ | A=(0,0); | ||
+ | B=(50,0); | ||
+ | M=(35,0); | ||
+ | N=(15,0); | ||
+ | |||
+ | draw(A--B); | ||
+ | |||
+ | dot(A,linewidth(4)); | ||
+ | dot(B,linewidth(4)); | ||
+ | dot(M,linewidth(4)); | ||
+ | dot(N,linewidth(4)); | ||
+ | |||
+ | label("$A$",A,dir(90)); | ||
+ | label("$B$",B,dir(90)); | ||
+ | label("$M$",M,dir(90)); | ||
+ | label("$N$",N,dir(90)); | ||
+ | </asy> | ||
+ | |||
+ | Note that <math>AN+BM = AB - MN = 30</math>. By AM-GM, the value of <math>AN\cdot BM</math> is at a maximum when <math>AN = BM = 15</math>. Thus, <math>AM = AN+MN = \boxed{35}</math>. | ||
+ | |||
+ | ~Sedro |
Revision as of 14:50, 9 September 2025
Problem
Let and
be points such that
. Points
and
lie on
such that
lies between points
and
and
lies between points
and
. Given that
and
is maximized, find the length of
.
Solution
Note that . By AM-GM, the value of
is at a maximum when
. Thus,
.
~Sedro